« first day (2684 days earlier)      last day (1794 days later) » 

8:12 AM
I don't understand the reopening of If $\sin(18^\circ)=\frac{a + \sqrt{b}}{c}$, then what is $a+b+c$? Could someone explain?
 
@YuiToCheng Once you see the third reopen-voter, who reopen it unilaterally due to a gold badge earned from answering PSQs in that area, you should know why...
 
and who also answered that question and got 5 upvotes
 
But since that question shows (some) effort, I do not object to that user answering it.
 
Agree the question is well-written... I just want to know his/her rationale behind the reopening. Otherwise, it seems an abuse of gold badges to me.
 
You can invite them to chat
Some other users with gold badges also do that
 
8:26 AM
@YuiToCheng You probably haven't seen much of the rampant abuse of privileges by PSQ answerers.
@TheSimpliFire And it's almost never productive to invite them to chat.
 
@user21820 If @YuiToCheng has time to kill
 
Well, that's up to Yuito, and I shouldn't say more.
 
OK, I will leave a comment asking for the explanation anyway...
 
@YuiToCheng By the way, I disagree that this is a PSQ; the phrasing in "it might seem reasonable to expect [...] Is this correct, and if so, can [...] be given?" suggests that the question is motivated largely by curiosity and not undeserving reasons (e.g. cheating at homework). I'd rather have 10 of these than 1 of the PSQs that I regularly post in here. =)
 
8:55 AM
@user21820 This is the "initial question" I'm referring to... All that follows are added by Robjohn
3
 
@YuiToCheng Oh okay thanks I followed the earlier link you provided and it had been beefed up, so I didn't know that.
 
 
1 hour later…
10:04 AM
@user21820 I will just point out that one of those posts has an entry in the reopen request thread.
 
10:16 AM
@MartinSleziak Thanks! CC @XanderHenderson @JoséCarlosSantos @RRL
D1, D2, D3.
D4, D5, D6.
D7, D8, D9.
And of course, this and this are pieces of pure nonsense. @JoséCarlosSantos @RRL @XanderHenderson @JyrkiLahtonen
 
10:43 AM
The last two are both gone @user21820
 
@JyrkiLahtonen Thanks! =)
 
 
2 hours later…
12:47 PM
C1, C2, C3.
C4, C5, C6.
@quid @AlexanderGruber: I can't believe this is still happening, right on the front page...
 
1:22 PM
@user21820 I closed it.
 
I love this question:
"Normally, I would just cheat. But Wolfram Alpha is giving me the wrong answers!"
 
1:34 PM
@quid Please undelete this. The OP removed the question immediately after an answer is given.
 
@quid Thanks!
 
[This seems like a rather minor edit to make, in light of all of the other problems with that question. :](math.stackexchange.com/posts/3231780/revisions)
 
@XanderHenderson Lol. Weird things happen.
 
@TheSimpliFire @user21820 Just a friendly reminder that we should probably focus on posts, not people.
(for what it's worth)
I should have linked directly to the edit, now that I have made more modifications: math.stackexchange.com/revisions/3231780/2
:\
 
1:51 PM
@XanderHenderson Yep. It was in response to the question "could someone explain"
 
2:28 PM
Does someone know how to refer to a system that consists of several equations in IEEE? For example i have a system described by equation (1) and a controller for system (1) described by equation (2). I basicaly want to refer to the whole dynamics (1) and (2). I was thinking if i could say: Dynamics (1,2).
 
@hola This is a chatroom for administrative stuff on MSE; you'd have better luck with these questions in the Mathematics chat
 
@hola This room is for discussing the closure, reopening, an undeleiton...
yeah... what @TheSimpliFire said...
 
got it. Thanks!
 
Oh, dear:
0
Q: Is this a valid proof of the parallel postulate?

Haziq MuhammadLet x1 be a line with a slope of tan(n) where 0 ≤ n < π/2. x1 = tan((π/2)-n)y, 0 < n ≤ π/2 Let y2 be a line with a slope of tan(m) where -π/2 < m ≤ 0; that has a y-intercept at b. x2 = tan((π/2)+m)y + b, -π/2 ≤ m < 0 Can we use systems of equations to prove that there will be a soluti...

 
@XanderHenderson I stopped reading when they said that n = pi/2 when the condition was 0 <= n < pi/2
 
2:51 PM
@YuiToCheng done
 
@quid Thanks!
 
3:27 PM
[ SmokeDetector | MS ] Few unique characters in answer, no whitespace in answer, repeating characters in answer (265): Inequality for $a,b,c>0$ $\sum_{cyc}\sqrt{\frac{a^3}{14a^2+4b^2}}\leq \sum_{cyc}\sqrt{\frac{a+b}{36}}$ by FatsWallers on math.SE
 
3:38 PM
This must be the fastest question I've seen so far on MSE to reach 10+ upvotes...
 
@YuiToCheng Multiple accounts perhaps, but that's only my personal suspicion
Usually wordy questions like this one (which is also low quality) don't attract many upvotes.
 
I think mathematicians hardly pay attention to the NBA...
 
4:16 PM
@YuiToCheng Why would you think that? I don't see any reason to believe that mathematicians would be any less interested in the NBA than anyone else.
For example, a couple of colleagues of mine are trying to build a better algorithm for playing fantasy sports (including basketball).
 
4:33 PM
@XanderHenderson Yeah, that's simply my prejudice. Ignore it.
 
 
1 hour later…
 
6 hours later…

« first day (2684 days earlier)      last day (1794 days later) »